LSAT and Law School Admissions Forum

Get expert LSAT preparation and law school admissions advice from PowerScore Test Preparation.

 Administrator
PowerScore Staff
  • PowerScore Staff
  • Posts: 8915
  • Joined: Feb 02, 2011
|
#40384
Complete Question Explanation
(The complete setup for this game can be found here: lsat/viewtopic.php?t=13080)

The correct answer choice is (D)

This question stem establishes that S is shown earlier than Q. All three of our templates allow for this sequence to occur, providing three solutions to the game:
  • 1. ..... If Q is third, then S must be first. This forces R into the second position, and T—in the fifth. The remaining ..... ..... variable, V, must be fourth:
PT72_Game_#2_#9_diagram 1.png
  • 2. ..... If V is third, and S must be shown earlier than Q, there are two solutions possible—depending on whether R ..... ..... is first or second:
PT72_Game_#2_#9_diagram 2.png
PT72_Game_#2_#9_diagram 3.png
It is evident from all three solutions that T must be shown fifth, which validates answer choice (D).

Answer choice (A) is incorrect, because Q can also be shown third.

Answer choice (B) is incorrect, because R can also be shown first.

Answer choice (C) is incorrect, because S can also be shown second.

Answer choice (E) is incorrect, because V can also be shown fourth.
You do not have the required permissions to view the files attached to this post.

Get the most out of your LSAT Prep Plus subscription.

Analyze and track your performance with our Testing and Analytics Package.